Calculus 1 finding absolute max and min

Click For Summary
The discussion revolves around calculating the absolute maximum and minimum values of the shuttle's acceleration during its liftoff phase. The velocity model provided is v(t)=0.001397167t^3−0.080965t^2+16.02t−0.39, and the derivative is v'(t)=0.004191201t^2-0.16193t+16.02. The student encountered complex solutions when applying the quadratic formula to find critical points, indicating no real solutions exist for v'(t) = 0. Consequently, the maximum and minimum values of v(t) should be determined by evaluating the endpoints of the interval from t = 0 to t = 79.2 seconds. The constant term was clarified to be 0.39, correcting a potential misunderstanding in the model.
Wm_Davies
Messages
51
Reaction score
0
A University of Rochester student decided to depart from Earth after his graduation to find work on Mars. Before building a shuttle, he conducted careful calculations. A model for the velocity of the shuttle, from liftoff at t = 0 s until the solid rocket boosters were jettisoned at t = 79.2 s, is given by

v(t)=0.001397167t3−0.080965t2+16.02t−039

(in feet per second). Using this model, estimate
the absolute maximum value
and absolute minimum value (I found this through using Wolfram Alpha I don't understand how I get it though which is a problem)
of the acceleration of the shuttle between liftoff and the jettisoning of the boosters.


I got the derivative to be

v'(t)= 0.004191201t2-0.16193t+16.02

I imagine that I would have to find the critical points of this to get the max and min

So I used the quadriatic formula but I am getting imaginary numbers when I do this so I am having trouble coming up with the absolute maximum and minimum any help would be appreciated.
 
Physics news on Phys.org
I assume you're looking for the absolute maximum value of the velocity.

For v(t), why do you have 039 as the constant term? Is that 39 or is there a decimal point missing in it?

Your v'(t) looks about right, and I find also that the solutions to v'(t) = 0 are complex, meaning that there are no real solutions to v'(t) = 0. This means that v'(t) > 0 for all t or that v'(t) < 0 for all t (not likely).

Since there are no times for which v'(t) = 0, to find the maximum and minimum values of v(t), check the endpoints of your domain, which is implied in your problem description.
 
it is supposed to be 0.39 instead of 39. I will try that what you suggest.
 
Question: A clock's minute hand has length 4 and its hour hand has length 3. What is the distance between the tips at the moment when it is increasing most rapidly?(Putnam Exam Question) Answer: Making assumption that both the hands moves at constant angular velocities, the answer is ## \sqrt{7} .## But don't you think this assumption is somewhat doubtful and wrong?

Similar threads

Replies
2
Views
2K
  • · Replies 5 ·
Replies
5
Views
1K
  • · Replies 4 ·
Replies
4
Views
2K
  • · Replies 10 ·
Replies
10
Views
3K
Replies
1
Views
2K
  • · Replies 2 ·
Replies
2
Views
4K
  • · Replies 4 ·
Replies
4
Views
2K
  • · Replies 9 ·
Replies
9
Views
3K
  • · Replies 10 ·
Replies
10
Views
3K
  • · Replies 5 ·
Replies
5
Views
2K